This graph shows a proportional relationship.
What is the constant of proportionality?

Answers

Answer 1

Answer:

[tex]m = \frac{3}{4}[/tex]

Step-by-step explanation:

Given

See attachment

Required

The constant of proportionality

Select two points on the graph

[tex](x_1,y_1) = (0,0)[/tex]

[tex](x_2,y_2) = (4,3)[/tex]

So, the constant of proportionality (k) is:

[tex]m = \frac{y_2 - y_1}{x_2 - x_1}[/tex]

[tex]m = \frac{3-0}{4-0}[/tex]

[tex]m = \frac{3}{4}[/tex]

This Graph Shows A Proportional Relationship.What Is The Constant Of Proportionality?

Related Questions

What is the value of x?

Answers

Answer:

D. 30°

Step-by-step explanation:

By exterior angle theorem:

x° + 70° = 100°

x° = 100° - 70°

x° = 30°

x = 30

There are 35 times as many students at Wow University as teachers. When all the students and
teachers are seated in the 8544 seat auditorium, 12 seats are empty. How many students attend
Wow University?

Answers

Given:

There are 35 times as many students at Wow University as teachers.

When all the students and teachers are seated in the 8544 seat auditorium, 12 seats are empty.

To find:

The total number of students.

Solution:

Let x be the number of teachers at Wow University. So, the number of student is :

[tex]35\times x=35x[/tex]

When all the students and teachers are seated in the 8544 seat auditorium, 12 seats are empty.

[tex]x+35x=8544-12[/tex]

[tex]36x=8532[/tex]

[tex]x=\dfrac{8532}{36}[/tex]

[tex]x=237[/tex]

The number of total students is:

[tex]35x=35(237)[/tex]

[tex]35x=8295[/tex]

Therefore, the total number of students is 8295.

find the slope of the line passing through the points (-2,5) and (3/2,2)

Answers

Answer:

slope = - [tex]\frac{6}{7}[/tex]

Step-by-step explanation:

Calculate the slope m using the slope formula

m = [tex]\frac{y_{2}-y_{1} }{x_{2}-x_{1} }[/tex]

with (x₁, y₁ ) = (- 2, 5) and (x₂, y₂ ) = ([tex]\frac{3}{2}[/tex], 2)

m = [tex]\frac{2-5}{\frac{3}{2}-(-2) }[/tex]

   = [tex]\frac{-3}{\frac{3}{2}+2 }[/tex]

   = [tex]\frac{-3}{\frac{7}{2} }[/tex]

  = - 3  × [tex]\frac{2}{7}[/tex]

  = - [tex]\frac{6}{7}[/tex]

which describes the graph of y=-(x+5)^2+3

Answers

Answer:

Maximum at (-5, 3)

Step-by-step explanation:

The quadratic equation is in vertex form, which specifically shows the vertex.

Since the a is negative, you get a maximum y value of 3.

Vertex form: y=a(x-h)^2+k

Maximum at (-5, 3)
Y’ = -2(x+5)
=> maximum when y’ = 0

Heya Kitties!
What is the value of x?

18x−16=−12x−4



x =

Answers

Answer:

2/5

Step-by-step explanation:

18x+12x=-4+16

30x=12

x=2/5

Answer:

18x-16=-12x-4

18x+12x=-4+16

30x=12

x=12/30

x=2/5

How do u solve this ?

Answers

You want to distribute the x

Choose the smallest number 3 1/8 or 10/3

Answers

Answer:

10/3

Step-by-step explanation:

31/8= 3.8

10/3= 3.3

jordan wants to save to buy a car and decides to open a banking account that is offering an interest rate of 4.5% compounded annually how much will jordan have in the account after 5 years it he deposits $7,000 today?

Answers

Answer:

8,723.27$

Step-by-step explanation:

Nathan is 1.55 meters tall. At 1 p.m., he measures the length of a tree's shadow to be 38.15 meters. He stands 32.9 meters away from the tree, so that the tip of his shadow meets the tip of the tree's shadow. Find the height of the tree to the nearest hundredth of a meter .

Answers

Answer:

11.26 m

Step-by-step explanation:

The height of the tree is about 11.25 meters.

What are similar triangles?

When the respective sides are proportional and the corresponding angles are congruent, two triangles are said to be similar.

Given that, the height of the person is 1.55 meters, the length of the tree's shadow is 38.15 meters, and the distance between the person and the tree is 32.9 meters.

Let the height of the tree be x.

Note that the scenario makes two similar triangles.

Since the ratio of the side lengths of similar triangles is proportional, it follows:

(38.15 - 32.9)/1.55 = 38.15/x

5.25/1.55 = 38.15/x

3.39 = 38.15/x

x = 38.15/3.39

x = 11.25

Hence, the height of the tree is about 11.25 meters.

Learn more about similar triangles: https://brainly.com/question/25882965

#SPJ2

A cinema is doing a promotion to celebrate their 50th anniversary for 1 week. They give

away a free drink to every 98th customer, a free bag of popcorn to every 112th customer and

a free cinema ticket to every 224th customer. Which lucky customer will be the first to

receive all 3 items?​

Answers

Answer:

1,568 customer

Step-by-step explanation:

Find the lowest common multiple of 98, 112, and 224

98 = 98, 196, 294, 392, 490, 588, 686, 784, 882, 980, 1078, 1176, 1274, 1372, 1470, 1568, 1666

112 = 112, 224, 336, 448, 560, 672, 784, 896, 1008, 1120, 1232, 1344, 1456, 1568, 1680, 1792, 1904

224 = 224, 448, 672, 896, 1120, 1344, 1568, 1792, 2016, 2240

The lowest common multiple of 98, 112, and 224 is 1568

Therefore, the 1,568th customer will be the first to receive all 3 iitem

Select the expression that represents the following statement: add 24 to the quotient of 16 and 8.

Answers

Answer:

16/8 + 24

Step-by-step explanation:

Question
The quotient of a number and 5 has a result of 2. What is the number?

Answers

Answer:

10.

Step-by-step explanation:

Answer:

The number is 10.

Step-by-step explanation:

x/5 = 2

Multiply both sides by 5.

5 * x/5 = 5 * 2

x = 10

Answer: The number is 10.

find the domain of f(x)=sec(2x)

Answers

Answer:

*Refer the image attached

Step-by-step explanation:

*Refer the image attached

At a certain store, a CD costs $12. If the cost of CDs were graphed as the output, compared to the number of
CDs purchased as input, which of the following would not be true of the graph?
A. The set of points would all lie on the same line.
B. The set of points would include the origin.
C. The set of points would rise from left to right.
D. The set of points would not graph a function.
Please select the best answer from the choices provided
A
B
C
D

Answers

Answer: The best and most correct answer among the choices provided by the question is A. The set of points would all lie on the same line.    

     

If a bus travel for 120 minutes at a speed of 75 kilometers per hour how far has the bus traveled?

Answers

Answer:

150 km

Step-by-step explanation:

Put the minutes into hours 120min is 2 hours.

Distance = speed * time

Distance = 75 * 2

Distance = 150

Answer:

150 kilometers

Step-by-step explanation:

if the bus is going 75 kilometers an hour and they traveled for 120 minutes (exactly 2 hours) then you would just multiply 75 by 2 to get 150 kilometers total.

Find the height h of the triangle.

Answers

Answer:

A, 12 times the square root of 3

Step-by-step explanation:

24 sqaured plus 12 squared=432

square root of 432=20.9

12 times the sqaure root of 3=20.9

The cost for 2 adults and 5 children to eat at the local buffet restaurant is \$46.50 . The cost for 1 adult and 1 child is $15. What is the cost of a child's meal ?

Answers

Answer:

c = 5.5

Step-by-step explanation:

2 a + 5 c = 46.50

a + c = 15

a = 15 - c

2(15-c ) +5c = 46.50

30 - 2c +5c = 46.5

3c = 16.5

c = 5.5

Let a be the adults and c be the children.

For the first equation there are 2 adults and 5 children so the equation is:-

2a + 5c = 46.50

And for the second equation there is 1 adult and 1 child. So the equation is:-

a + c = 15

a = 15 - c

You take this formula and substitute it to the above formula (2a + 5c = 46.50)

➡️ 2a + 5c = 46.50

➡️ 2(15 - c) + 5c = 46.50

➡️ 30 -2c + 5c = 46.50

➡️ 5c - 2c = 46.50 - 30
(You collect like terms)

➡️ 3c = 16.5

➡️ c = 16.5/3

➡️ c = 5.5

Plot point C in GeoGebra to verify that it indeed lies on AB. You can enter the coordinates in the Input window to plot the point . Also , verify the distances you calculated in question 3 using GeoGebra . Take a screenshot showing the distances displayed by GeoGebra , and paste it below .

Answers

Answer:

The picture of file

Step-by-step explanation:

Your'e welcome

What is the value of k?

Answers

Answer:

Step-by-step explanation:

recall that the inside angles of a triangle add up to 180°

and also that the angle 115° is the other part of a 180° total

180=115+y

180-115 =65

then the inside angle at Y  is 65°

add the three inside (interior of the triangle) angles up

180= 65 + (4k+5) + (6k+10)

180 = 65 + 10k + 15

180 = 80 + 10k

100  = 10k

10 = k

see how I did that?  

On the unit circle, which of the following angles has the terminal point
coordinates.
A. 45
B. 135
C. 225
D. 315

Answers

Answer: C. 225

Step-by-step explanation:

The center of the circle is at the point
, and its radius is
units. The equation of this circle in standard form is
.

Answers

Is there a picture that goes with this?

Hello, Brainly community!

This question is for all of those Calculus people out there.

The volume of a swimming pool is changing with respect to time, such that the volume is given by W(t), where W(t) is measured in cubic centimeters and t is measured in seconds. A tangent line is shown for W(t) at t = 3 seconds. Determine the best estimate for the value of the instantaneous rate of change of W(t) when t = 3.
(I've narrowed down the answer choices to 2, and just really need to find the right way of thinking to find the answer)

(A) W(lim t) as t goes to 3.
(B) [W(3.1) - W(2.9)] / 0.2.

Thank you in advance!

Answers

Answer:

(B)  [tex]\displaystyle \frac{W(3.1) - W(2.9)}{0.2}[/tex]

General Formulas and Concepts:

Calculus

Limits

Derivatives

The definition of a derivative is the slope of the tangent line.

Derivative Notation

Instantaneous Rates

Tangent Line: [tex]\displaystyle f'(x) = \frac{f(b) - f(a)}{b - a}[/tex]

Step-by-step explanation:

Since we are trying to find a rate at which W(t) changes, we must find the derivative at t = 3.

We are given 2 close answer choices that would have the same numerical answer but different meanings:

(A)  [tex]\displaystyle \lim_{t \to 3} W(t)[/tex](B)  [tex]\displaystyle \frac{W(3.1) - W(2.9)}{0.2}[/tex]

If we look at answer choice (A), we see that our units would simply just be volume. It would not have the units of a rate of change. Yes, it may be the closest numerically correct answer, but it does not tell us the rate at which the volume would be changing and it is not a derivative.

If we look at answer choice (B), we see that our units would be cm³/s, and that is most certainly a rate of change. Answer choice (B) is also a derivative at t = 3, and a derivative tells us what rate something is changing.

∴ Answer choice (B) will give us the best estimate for the value of the instantaneous rate of change of W(t) when t = 3.

Topic: AP Calculus AB/BC (Calculus I/I + II)

Unit: Differentiation

Book: College Calculus 10e

5) On each birthday Rosa gets as many roses as she is old in years. She still has all the dried flowers and there are now 120 of them. How old is she? A) 10 B) 12 C) 14 D) 15 E) 20

Answers

Answer:

D) 15

Step-by-step explanation:

This is an arithmatic progression.

The formula for the sum of arithmatic progression is

[tex]s = \frac{n}{2} (2a + (n - 1)d)[/tex]

where d is the common difference between successive terms and a is the first term. By applying this formula,

[tex]120 = \frac{n}{2} (2(1) + (n - 1)(1)) \\ 120 = \frac{n}{2} (1 + n) \\ n(1 + n) = 240 \\ n {}^{2} + n - 240 = 0 \\ (n - 15)(n + 16) = 0 \\ n = 15 \: or \: n = - 16(reject)[/tex]

According to the number line, what is the distance between points A and B?

0 6 units
7 units
O 12 units
O 14 units

Answers

Answer:

14 units

Step-by-step explanation:

A = - 2, B = 12

Therefore,

d(A, B) = 12 - (-2) = 12 + 2 = 14 units

What is the scale of the y-axis in this coordinate graph?


A. 1 tick mark represents 1 unit
B. 1 tick mark represents 8 units
C. 1 tick mark represents 12 units
D. 1 tick mark represents 16 units

Answers

Answer:

Obviously B

Step-by-step explanation:

Which operation will solve the following word problem? Jeff earns $14.00 per hour, Tom earns half as much as Jeff. How much does Tom earn per hour?


Multiplication


Subtraction


Addition


Division

Answers

Answer:

The correct option is (d).

Step-by-step explanation:

Given that,

Jeff earns $14.00 per hour.

Tom earns half as much as Jeff.

We need to find the amount earn by Tom per hour.

Tom's amount = Jeff's amount/2

So,

[tex]T=\dfrac{14}{2}\\\\T=\$7[/tex]

So, Tom earn $7 per hour. Hence, division operation is used. Jeff's amount is divided by 2.

solve for x. Round to the nearest tenth, if necessary.​

Answers

Answer:

7.1

Step-by-step explanation:

We used SOHCAHTOA because it's a right angle triangle

So because we have an angle with an adjacent of 6.3 and hypotenuse of x

We will use

Cos=adjacent /hypotenuse

7.1 like the last answer if not you can round that

What type of health screening would this patient most likely receive?
Sue is a 45-year-old woman with a family history of breast cancer. Her healthcare professional will most likely recommend that she receive a .

Answers

Answer:

A mammogram is what she would receive

Step-by-step explanation:

Find value of x.
A. 110
B. 47
C. 68
D. 112

Answers

Answer:

B

Step-by-step explanation:

The sum of the inner angles of a quadrilateral is 360 degrees

135 + 110 + 68 + x = 360

313 + x = 360

x = 47 degrees

Answer:

47

Step-by-step explanation:

whole thing is 360 degrees

68 + 110 + 135 = 313

360 - 313 = 47

x looks small too (if you had to guess in a multiple choice question)

Decrease £19064.67 by 9.5%

Give your answer rounded to 2 DP. (decimal place)

Please Help !!!

Answers

Answer:

17253.52

Step-by-step explanation:

100-9.5=90.5

19064.67*0.905=17253.52

Other Questions
In sentence 11, the compound adjective modifying the wordcommittee is incorrectly hyphenated. Which revision corrects theerror?Select one:a. cultural/affairs committeeO b. cultural affairs-committeeO c. cultural-affairs-committeeO d. cultural-affairs committee How did the city of Venice help begin the age of exploration? A. They built new ships to help explorers. B. Merchants began to trade with the Middle East. C. Armies from Venice conquered Asian countries. D. Crusaders conquered the city of Jerusalem.Which two European countries initiated (started) the race for a water route to Asia? A. France and England B. Portugal and Holland C. Spain and England D. Portugal and SpainWhich event was a result of the other three? A. Crusades B. Dark Ages C. Age of Exploration D. RenaissanceWhich continent were European explorers trying to reach in order to find silk, spices, rugs, and perfumes? A. South America B. Asia C. North America D. EuropeHow did Spain benefit economically from Columbus voyages to the New World? A. Spain gained new people to add to its armies in Europe. B. Spain learned about new ways to govern its people. C. Spain gained vast wealth from gold and slaves. D. Spain gained converts to the Christian religion. Reliable Industries is a maker of component parts in heating and cooling ventilation systems. The company is looking for a site on which to build a new production facility. Major considerations the company must take into account are that any area selected has adequate rail access and is at the hub of major highways to quickly move products. Reliable is in the process of There are 3 boxes on stage that appear identical, but one is Lucky. The boxes are full of tickets; some are labeled "win" and the others are labeled "lose." In the Lucky box, ninety percent of the tickets are winners. In each of the other two boxes, only twelve percent of the tickets are winners.1. You will pick a box at random and draw one ticket from it at random.2. What is the probability you will draw a winning ticket? 3. If you do draw a winning ticket, what is the chance it came from the Lucky box? Oliver's long-term care policy covers only services in a nursing facility and pays nothing for services provided at home or in the community. What kind of LTC policy does Oliver own? Pour aller . supermarch, sil vous plat ?A) laB) auC) do anyone have total english icse class 9 solutions Question in the picture Professor Hanoi's research efforts focus on how the use of heuristics influences the way people think about and assess financial risks. Which specialty area does his research best represent if my thoughts are correct i think ill mark some1 barinliest if they get this answer correct!!?Mi nombre es Juana y trabajo en mi oficina desde las ocho de la maana a las cinco de la tarde. Uso mi computadora y me gusta mucho trabajar con nmeros.According to the text, Juana ________. es abogada es contable es comadrona es agricultora What does the underlined word mean in the following sentence?What does the underlined word mean in the following sentence?Mi padre vende muchas verduras en el mercado.applespeachesvegetablesdrinks What is an electric fuse? What is the working principle of electric fuse? Which equation represents a line that passes through (2,-) and has a slope of 3?Oy-2 = 3(x + 2)Oy - 3 = 2(x+)Oy+ 1 = 3(x - 2)Oy+ < = 2(x-3)Help? A(n)____is the product of gametes from two differentsources. Can someone please be generous & help Ive been struggling all night 15. Ilang lalawigan ang bumubuo sa Gitnang Mindanao? Pathos refers to the credibility of a speaker.TrueFalse The weight of a body is 600 N. What is the mass of the body on the surface of the earth? Wat weet jy omtrent die title tarental spesifiek saam met die woordjie saam True or false: Boron contains 2s22p1 valence electrons, so only one p orbital is needed to form molecular orbitals.